Post-UTME Past Questions - Original materials are available here - Download PDF for your school of choice + 1 year SMS alerts
Project Topics and Materials, Post-UTME past questions - Download now or upload yours to get paid

Physics Past Questions

Clear Selections
Change Subject Post a Question Check Syllabus Study My Bookmarks Past Questions Videos Watch Video Lessons Download App

Project Topics and Materials, Post-UTME past questions - Download now or upload yours to get paid
NECO offline past questions - All questions, answers & explanations in one app 25712
Join your school's WhatsApp group
4431

 

Open photo

 

 

You are provided with a metre rule, lens, screen, ray box, and other necessary apparatus.

i. Set up the experiment as shown in the diagram above. Measure and record the diameter a\(_{0}\), of the illuminated object.

ii. Place the object at a distance x= 25cm from the lens. Adjust the screen until a sharp image is obtained on the screen.

iii. Measure and record the diameter, a, of the image.

iv. Measure and record the distance v between the lens and the screen.

v. Evaluate y = P = \(\frac{1+y^{2}}{y}\) and T= x+v.

vi. Repeat the procedure for x = 30cm, 35cm, 40cm and 45cm. In each case, determine the corresponding values of a,v,y, P and T.

vii. Tabulate your results.

viii. Plot a graph of P on the vertical axis against T on the horizontal axis starting both axes from the origin (0,0).

ix. Determine the slope, s, of the graph.

x. Determine the intercept, c, on the horizontal axis.

xi Evaluate K = \(\frac{c}{2}\)

xii. State two precautions taken to ensure accurate results.

(b)i.  Explain the statement, the focal length of a converging lens is 20cm.

ii. An object is placed at a distance x from a converging lens of focal length 20cm. If the magnification of the real image is 5, calculate the value of x.

View Answer & Discuss WAEC 2018
4432

 

Open photo

You are provided with an ammeter, resistor, key, metre bridge, and other necessary apparatus.

i. Connect a circuit as shown in the diagram above.

ii. Close the key and use the jockey to make contact with AB at N such that AN = d = 25cm

iii. Read and record the ammeter reading.

iv. Evaluate 1\(^{-1}\).

v. Repeat the procedures for values of d = 35cmm, 50cm, 65cm and 80cm. In each case, record I and determine 1\(^{-1}\)

vi. Tabulate your results.

vii. Plot a graph with log on the vertical axis and d on the horizontal axis.

viii. Determine the slope, s of the graph.

ix. State two precautions taken to obtain accurate results.

(b)i. Use your graph to determine the value of d = l = 1.5A.

ii. State two factors that affect the resistance of a wire.

View Answer & Discuss WAEC 2018
4433

Open Photo

You are provided with two retort stands, two-metre rules, pieces of thread and other necessary apparatus.

i. Set up the apparatus as illustrated above ensuring the strings are permanently 10cm from either end of the rule.

ii. Measure and record the length L = 80 cm of the two strings.

iii. Hold both ends of the rule and displace the rule slightly, then release so that it oscillates about a vertical axis through its centre.

iv. Determine and record the time t for 10 complete oscillations.

v. Determine the period T of oscillations.

vi. Evaluate log T and L.

vii. Repeat the procedure for four other values of L= 70 cm, 60 cm, 50 cm, and 40 cm

viii. Tabulate your readings.

ix. Plot a graph with log T on the vertical axis and log L on the horizontal axis.

x. Determine the slope, s, and the intercept, c on the vertical axis.

xi. State two precautions taken to ensure accurate results. 

(b)i. Define simple harmonic motion.

ii. Determine the value of L corresponding to t= 12 s from the graph in 1.

View Answer & Discuss (2) WAEC 2017
NECO offline past questions - All questions, answers & explanations in one app 25712
Project Topics and Materials, Post-UTME past questions - Download now or upload yours to get paid
Join your school's WhatsApp group
4434

Open Photo

You are provided with a beaker, a thermometer, a stirrer Stopwatch/clock, measuring cylinder, table salt, water, and other necessary materials.

i. You Measure 200cm\(^{3}\) of water into the beaker.

ii. Heat the water until it boils steadily for about 2 minutes.

iii. Read and record the boiling point b\(_{0}\).

iv.  Add table salt of mass M = 10.0 g to the boiling water and stir continuously until another boiling point b\(_{1}\) is attained.

v. Read and record b\(_{i}\).

vi. Evaluate \(\theta_{i}\) = (b\(_{i}\) - b\(_{0}\))

vii. Using the same mixture, repeat the procedure four more times by adding 10.0 g of salt each time to give the cumulative mass M\(_{i}\) of salt as 20 g, 30g, 40g, and 50g.

viii. In each case allow the mixture to boil steadily for at least 2 minutes then read and record the boiling point b. 

ix. Tabulate your readings.

x. Plot a graph with M\(_{i}\) on the vertical axis and \(\theta_{1}\) on the horizontal axis.

xi. Determine the slope, s, of the graph.

xii. State two precautions taken to ensure accurate results.

(b) i. Define the boiling point of a liquid.

ii. What effects do impurities have on the boiling point of a liquid?

View Answer & Discuss WAEC 2017
4435

Open Photo

You are provided with cells, a potentiometer, an ammeter, a voltmeter, a bulb, a key, a jockey, and other necessary materials.

i. Measure and record the e.m.f E of the battery.

ii. Set up a circuit as shown in the diagram above.

iii. Close the key K and use the jockey to make a firm.

iii. Contact at J on the potentiometer wire such that PJ = x= 10cm.

iv. Take and record the voltmeter reading V and the () Corresponding ammeter reading.

v. Evalute log V and log I.

vi. Repeat the procedure for five other values of x = 20 cm, 30 cm, 40 cm, 50 cm, and 60 cm.

vii. Tabulate your readings.

viii. Plot a graph with log I on the vertical axis and log V on the horizontal axis.

ix. Determine the slope s, of the graph.

x. Determine the intercept, c, on the vertical axis.

xi. State two precautions taken to ensure accurate results.

(b)i. How is the brightness of the bulb affected as x increases?

ii. List two electrical devices whose actions do not obey Ohm's law.

View Answer & Discuss WAEC 2017
Start a Free Practice Test
 
Join your school's WhatsApp group
Post-UTME Past Questions - Original materials are available here - Download PDF for your school of choice + 1 year SMS alerts
Project Topics and Materials, Post-UTME past questions - Download now or upload yours to get paid